¿Cómo implican exactamente las trayectorias Regge lineales la estabilidad?

(para una versión más confusa, consulte physics.stackexchange: ¿Qué pasa con el argumento de Mandelstam de que solo las trayectorias regulares lineales son estables? )

Hay un argumento de 1974 de Mandelstam de que las trayectorias lineales de Regge implican estabilidad, de "Dual-Resonance Models" de 1974, sciencedirect.com/science/article/pii/0370157374900349 . Ampliar la función de trayectoria Regge α ( s ) en una relación de dispersión con dos restas:

α ( s ) = b + a s + 1 i π 0 I metro ( α ( s ) ) s s d s

La parte imaginaria de α ( s ) da el decaimiento de los estados de la cuerda, ya que donde toca un número entero te dice dónde están los polos. Entonces, si las resonancias de las cuerdas son exactamente estables, entonces la parte imaginaria es cero y la trayectoria es lineal.

Este argumento me molestó por estas razones:

  • Parece funcionar igual de bien con dos restas, tres restas, etc. ¿Puedes concluir que las trayectorias Regge exactamente cuadráticas o exactamente cúbicas también son estables? ¿Qué es una trayectoria cuadrática o cúbica?
  • La función de trayectoria de Regge aparece en el exponente, por lo que debe tomar un registro para extraerla. ¿Por qué es claro que tiene una representación como la anterior, sin una contribución de corte en s negativa?
  • En la teoría de cuerdas, las trayectorias son lineales cuando son de larga duración, pero la función de trayectoria no parece tan fundamental hoy en día. ¿Existe una formulación más moderna de esto, que le diga qué límites de cadena no interactúan solo por una condición en el espectro?

Mandelstam generosamente me envió un breve comentario por correo electrónico, diciendo esencialmente que la parte imaginaria de la función de trayectoria es una vida, y de hecho esto es obvio por el hecho de que da la posición de las resonancias, pero todavía estoy confundido con respecto a las preguntas anteriores.

Incluso una respuesta parcial sería apreciada.

Ron, estaba buscando cuerdas y trayectorias regge y surgió esta vieja pregunta tuya. Ha habido algunas discusiones sobre la validación de la teoría de cuerdas y me he estado preguntando si has descubierto alguna "predicción", comprobable con la gran cantidad de datos de resonancia. , que podría dar una teoría de cuerdas.
@annav: Este tipo de teoría de cuerdas ahora se conoce como AdS/QCD, y nadie está en desacuerdo en que es predictivo para las resonancias de hadrones, aunque se puede discutir el grado de concordancia cuantitativa. Este material predice cosas como simetría local oculta (teoría Hidden Regge en los años 80), bariones topológicos (física de grandes N de los años 80) y Polchinski-Tan Pomerons (pomerones BFKL en la teoría de cuerdas, principios de la década de 2000). Hay mucho trabajo, esto debería dar buenos resultados, pero AdS/QCD es el más reciente y el más directo. Esto no es lo mismo que las predicciones de la teoría fundamental de cuerdas, los resultados no son controvertidos.
gracias . quiere decir que es una validación de teoría de cuerdas de segundo nivel, validación mediante la incorporación de teorías más limitadas que en sí mismas son validadas por datos. es decir, ninguna teoría de cuerdas de línea directa que no haya sido ya predicha por los menores. :(

Respuestas (1)

No tengo ninguna respuesta clara, pero el argumento parece incompleto. Creo que sabemos en el infinito- norte C límite de QCD que tenemos resonancias exactamente estables y algunas trayectorias Regge casi lineales en alguna región, pero que no son perfectamente lineales y fallan gravemente en ser lineales en s negativas donde BFKL describe la física Regge. http://arxiv.org/abs/hep-th/0603115 por Brower, Polchinski, Strassler y Tan analiza este tipo de cosas con cierto detalle y podría señalar alguna literatura más antigua que tiene algo que decir.

La palabra "incompleto" es ambigua, probablemente quieras decir "incorrecto". Conozco el asunto de Brower Polchinski Strassler Tan, y su argumento vincula el BFKL perturbador con el pomeron no perturbador. Creo que la ubicación de la vuelta depende de N de una manera consistente con Mandelstam, de modo que en el límite puro de N grande, la trayectoria del pomeron es recta. Sin embargo, no estoy seguro, pero es bueno verificarlo. Gracias por la respuesta.